« Discussion utilisateur:Cgolds » : différence entre les versions

Le contenu de la page n’est pas pris en charge dans d’autres langues.
Une page de Wikiversité, la communauté pédagogique libre.
Contenu supprimé Contenu ajouté
Cgolds (discussion | contributions)
m →‎DTF : preuve de dtf donc impossibilité
Aucun résumé des modifications
Ligne 19 : Ligne 19 :
Autrement dit, comment démontrer qu'il est impossible de reconstituer une égalité à la puissance n sauf 2, en multipliant ces trois nombres par 4<sup>p</sup> et jouant sur les restes ? On peut le faire pour n = 2 [[Utilisateur:Supreme assis|<font color="darkslategray">Supreme assis</font>]] ([[Discussion Utilisateur:Supreme assis|<font color="darkslategray">''grain de sel''</font>]]) 19 janvier 2019 à 15:40 (UTC)
Autrement dit, comment démontrer qu'il est impossible de reconstituer une égalité à la puissance n sauf 2, en multipliant ces trois nombres par 4<sup>p</sup> et jouant sur les restes ? On peut le faire pour n = 2 [[Utilisateur:Supreme assis|<font color="darkslategray">Supreme assis</font>]] ([[Discussion Utilisateur:Supreme assis|<font color="darkslategray">''grain de sel''</font>]]) 19 janvier 2019 à 15:40 (UTC)
: Quand j’ai écrit que la preuve existait pour n=3, cela signifiait "la preuve du DTF", puisque c’est cela dont nous parlions, donc la preuve qu’il n’existe PAS de solutions entières positives strictement a, b, c telles que a^3+b^3=c^3 (alors qu’il en existe si on remplace 3 par 2). Comme vous l’a expliqué Marvoir, vous n’avez pas de preuve du DTF (impossibilité de solutions, donc) même dans ce cas. --[[Utilisateur:Cgolds|Cgolds]] ([[Discussion utilisateur:Cgolds|discussion]]) 19 janvier 2019 à 16:06 (UTC)
: Quand j’ai écrit que la preuve existait pour n=3, cela signifiait "la preuve du DTF", puisque c’est cela dont nous parlions, donc la preuve qu’il n’existe PAS de solutions entières positives strictement a, b, c telles que a^3+b^3=c^3 (alors qu’il en existe si on remplace 3 par 2). Comme vous l’a expliqué Marvoir, vous n’avez pas de preuve du DTF (impossibilité de solutions, donc) même dans ce cas. --[[Utilisateur:Cgolds|Cgolds]] ([[Discussion utilisateur:Cgolds|discussion]]) 19 janvier 2019 à 16:06 (UTC)
:: Mais oui, c'est cela. Il faut raisonner '''à l'envers''' (voir AREPO ↔ OPERA du carré SATOR). Autrement dit, on ne part pas de a, b, et c pour démontrer qu'on peut avoir a^n+b^n=c^n ... mais dire que cette configuration n'est possible que si on peut partir de <u>trois autres nombres</u> (par exemple 9 11 et 20) et qu'on peut arriver à une telle expression par un processus de x4^p + restes. J'ai compris ! je dis : si on trouve un processus qui fonctionne et produit le résultat souhaité pour n=2, eh bien, il ne marche que pour n=2. Je suppose qu'il existe au moins un quadruplet.
::Et vous dites : on cherche un quadruplet qui !!! voilà pourquoi on ne peut se comprendre. Ne nous énervons plus. Un grand merci, car j'ai fait plus de progrès en quelques échanges avec vous. Votre partie directe serait ma partie réciproque. Pas évident. Il faut que je réfléchisse.
::Tout est dans la phrase négative. « il n'existe pas ». Est-ce que la double négation peut donner une équivalence ? That is THE question. « ''Ce qui peut se traduire d'une manière équivalente par : Si ces trois nombres existent et vérifient cette relation, alors n N'EST PAS strictement supérieur à 2. Et alors, n = 1 OU n = 2.'' » ????. Bon week-end [[Utilisateur:Supreme assis|<font color="darkslategray">Supreme assis</font>]] ([[Discussion Utilisateur:Supreme assis|<font color="darkslategray">''grain de sel''</font>]]) 19 janvier 2019 à 17:09 (UTC)

Version du 19 janvier 2019 à 17:09

DTF

Si c'est une question de forme, vous pouvez (devez) m'aider.

Pour les nombres SIMPLEMENT-PAIRS et MULTI-PAIRS, vous pouvez infirmer ou confirmer la proposition :
Si n est un nombre simplement pair (resp multi-pair), alors pour tout k entier, n + 4k est simplement pair (resp multi-pair). Comment le diriez-vous plus adroitement SVP ? Supreme assis (grain de sel) 18 janvier 2019 à 16:15 (UTC)[répondre]

Notification Supreme assis :. Vos nombres simplement pairs sont ceux divisibles par 2 et pas par 4 (ils correspondent à ce qu’Euclide appelle impairement pairs), les multi-pairs sont ceux divisibles par 4 (les pairement pairs d’Euclide). 4k est toujours pairement pair (ou "multi-pair" comme vous dites). J’écrirais donc simplement : "si n est divisible par 2 et pas par 4 (resp. est divisible par 4), alors il en est de même pour n+4k, pour tout k". Cela suffit en principe. Si vous voulez aussi donner une preuve et qu’elle soit accessible au plus grand nombre : "Si n est divisible par 2 et pas par 4, il s’écrit n= 2n’, avec n’ impair. Donc n+4k s’écrit 2n’+4k= 2(n’+2k). Si n’ est impair, il en est de même pour n’+2k donc n+4k est divisible par 2, mais pas par 4. Pour le deuxième cas, si n est divisible par 4, il s’écrit n=4n’, donc n+4k=4(n’+k), ce qui montre que n+4k est aussi divisible par 4". Cordialement, --Cgolds (discussion) 18 janvier 2019 à 16:28 (UTC)[répondre]
Ah ! voilà une réponse qui me ravit à laquelle j'adhère ici. Ce malentendu tombe donc. Et d'un. Il reste à comprendre pourquoi le nombre de divisions possibles par 4 correspond à p et pourquoi on peut remonter à 48² + 55² = 73² à partir de 9 + 11 = 20, le seul nombre au carré étant 9. J'ai une petite idée. Je vous l'aurai bien soumise pour avoir votre avis, si votre temps le permet et si vous acceptez une discussion avec un néophyte, petit prof de maths à la retraite de 71 balais ? Supreme assis (grain de sel) 18 janvier 2019 à 17:16 (UTC)[répondre]
Le problème comme Marvoir l’a expliqué n’a jamais été dans le début de vos explications sur la parité, les divisions par 4, etc. Le premier problème est que vous n’avez pas n.i= 2k. Je suggère que vous fixiez n=3 et essayez votre démarche dans ce cas pour que vous voyez ce qui ne marche pas. --Cgolds (discussion) 18 janvier 2019 à 17:50 (UTC)[répondre]
Ah ! hélas, on ne peut pas et vous le savez bien. Voir ce paragraphe. Autrement dit, ce n'est qu'une proposition déterminante dont le résultat peut être vérifié logiquement (elle est cohérente !). Et le problème est qu'on ne peut pas apporter la preuve du contraire. Tant que les restes [mod4] sont égaux on conserve l'égalité et la puissance n du nombre pair. Même si ma façon de le dire n'est pas parfaitement académique, la réalité est là. J'aurais voulu avoir le temps de résoudre cette dernière énigme ou conclure que il existe des choses vraies qui sont indémontrables. En quel cas la Vérité se situerait bien dans l'espace logique défini par {ni-vrai ; ni-faux} qui est un espace intermédiaire entre l'observable et le non-observable. C'est là le projet de cette recherche à la frontière du « métaphysique » comme l'indique Lionel Scheepmans (d · c · b · s). Mon dernier espoir est que vous compreniez ici que personne ne peut conclure que ma démonstration est fausse sans apporter un élément d'absurdité ou une exception pour n = 3. C'est impossible. Et je suis le premier à le regretter, soyez-en assurée. Si vous m'avez permis de régler un problème avec les nombres impairement pairs, et je vous en remercie, il n'en reste pas moins que le pas de 4 est une structuration fondamentale sur les entiers. Laissez-moi terminer ceci SVP. Et j'aurais grand honneur à en discuter avec vous parce que vous pouvez voyager dans les espaces-temps.
En conséquence, je demande à JackPotte (d · c · b · s) la possibilité de réintégrer la page dans mon espace perso et de conserver la page de discussion dans la recherche. Je conçois parfaitement que je suis aux frontières du visible et de l'invisible et que je n'appartiens ni au monde visible, ni au monde invisible. Ma place n'est dans aucun monde (mathématique, philosophique, métaphysique, ésotérique, ...). Alors où est-elle ? Pourtant j'existe. J'ai confiance en vous et ma vie dépend de cette confiance. Vous êtes ma seule connexion de qualité.Cordialement Supreme assis (grain de sel) 19 janvier 2019 à 10:31 (UTC)[répondre]
La preuve pour n=3 existe, il ne s’agit donc pas d’un cas indémontrable. C’est pourquoi si vous annoncez une preuve de DTF, il faut pouvoir l’inclure (ou dire que la preuve ne s’applique qu’à certains n, par exemple). Je crois pour ma part que tant qu’une recherche n’aboutit pas, on ne peut pas l’inclure dans wikiversité qui est après tout un espace destiné à enseigner. Les chercheurs professionnels ne font pas autrement, y compris sur les espaces de recherche collaborative comme Math overflow : on peut y poser des questions, et des réponses précises, mais pas des idées sur des possibilités. C’est pourquoi j’ai corrigé un peu le début de votre présentation sur la parité, qui n’est pas fausse, pour la rendre un peu plus facile à lire ; mais la partie sur la preuve de DTF est autre chose, car cela peut induire une confusion. Je ne connais pas assez le fonctionnement de Wikiversité pour déterminer ce qu’il en est sur les pages personnelles. Cordialement, --Cgolds (discussion) 19 janvier 2019 à 12:01 (UTC) PS : Votre confiance me touche, mais je n’ai pas assez de disponibilité pour suivre systématiquement vos rédactions.[répondre]
Merci vraiment. Vous pouvez me donner l'expression a³ + b³ = c³ pour voir si la partie directe fonctionne aussi ? On essayera quand me de communiquer de temps en temps Supreme assis (grain de sel) 19 janvier 2019 à 13:13 (UTC)[répondre]
Je n’ai pas compris votre question. Il y a une expression pour les solutions de a^2 + b^2 = c^2 (a=d(p^2-q^2), b=2dpq, c=d(p^2+q^2), avec p et q entiers de parité différente et premiers entre eux et d entier quelconque), mais il n’y a pas d’expression analogue pour les cubes, puisqu’il s’agit de prouver qu’il n’y a pas de solution en entiers strictement positifs. --Cgolds (discussion) 19 janvier 2019 à 13:35 (UTC)[répondre]

Ah ! il me semblait bien. C'est certainement un malentendu, car vous avez écrit « La preuve pour n=3 existe ». Mais, alors, nous sommes parfaitement d'accord !!! Le processus de divisions successives qui débouche sur k = p, qui conserve la puissance n-ième du nombre pairement pair initial et qui n'est plus possible pour k > p EST UN EXCELLENT PROCESSUS que l'on peut poser en condition nécessaire puisque l'on sait que cela fonctionne pour n = 2, qu'il conserve l'égalité et la puissance n-ième du nombre pair. La question est donc bien d'établir la réciproque pour tout n, à savoir si la formulation de Marvoir (d · c · b · s) convient. A savoir est-ce que si on a an + B = C, on a B et C sont impairs à la puissance n dès lors que l'on multiplie par 4, p fois, en trafiquant les restes pour que l'égalité soit conservée ? Me fourvoyé-je encore ? Me trompé-je ? Ou sommes-nous prêts de nous rejoindre ?

Autrement dit, comment démontrer qu'il est impossible de reconstituer une égalité à la puissance n sauf 2, en multipliant ces trois nombres par 4p et jouant sur les restes ? On peut le faire pour n = 2 Supreme assis (grain de sel) 19 janvier 2019 à 15:40 (UTC)[répondre]

Quand j’ai écrit que la preuve existait pour n=3, cela signifiait "la preuve du DTF", puisque c’est cela dont nous parlions, donc la preuve qu’il n’existe PAS de solutions entières positives strictement a, b, c telles que a^3+b^3=c^3 (alors qu’il en existe si on remplace 3 par 2). Comme vous l’a expliqué Marvoir, vous n’avez pas de preuve du DTF (impossibilité de solutions, donc) même dans ce cas. --Cgolds (discussion) 19 janvier 2019 à 16:06 (UTC)[répondre]
Mais oui, c'est cela. Il faut raisonner à l'envers (voir AREPO ↔ OPERA du carré SATOR). Autrement dit, on ne part pas de a, b, et c pour démontrer qu'on peut avoir a^n+b^n=c^n ... mais dire que cette configuration n'est possible que si on peut partir de trois autres nombres (par exemple 9 11 et 20) et qu'on peut arriver à une telle expression par un processus de x4^p + restes. J'ai compris ! je dis : si on trouve un processus qui fonctionne et produit le résultat souhaité pour n=2, eh bien, il ne marche que pour n=2. Je suppose qu'il existe au moins un quadruplet.
Et vous dites  : on cherche un quadruplet qui !!! voilà pourquoi on ne peut se comprendre. Ne nous énervons plus. Un grand merci, car j'ai fait plus de progrès en quelques échanges avec vous. Votre partie directe serait ma partie réciproque. Pas évident. Il faut que je réfléchisse.
Tout est dans la phrase négative. « il n'existe pas ». Est-ce que la double négation peut donner une équivalence ? That is THE question. « Ce qui peut se traduire d'une manière équivalente par : Si ces trois nombres existent et vérifient cette relation, alors n N'EST PAS strictement supérieur à 2. Et alors, n = 1 OU n = 2. » ????. Bon week-end Supreme assis (grain de sel) 19 janvier 2019 à 17:09 (UTC)[répondre]